8

I need an asymptotic expansion/closed form for $$\sum_{k=1}^{\infty}\int_{0}^{\infty}(B(x+n+k,n+1))^2\ dx$$ where $B(m,n)$ is the Beta function and $n\in\mathbb{N}$.

Denote $$I_n=\sum_{k=1}^{\infty}\int_{0}^{\infty}(B(x+n+k,n+1))^2\ dx$$ By definition of Beta function $$I_n=\sum_{k=1}^{\infty}\int_{0}^{\infty}\left(\frac{\Gamma(x+n+k)\Gamma(n+1)}{\Gamma(x+2n+k+1)}\right)^2\ dx$$ $$I_n=(n!)^2\sum_{k=1}^{\infty}\int_{0}^{\infty}\left(\frac{\Gamma(x+n+k)}{\Gamma(x+2n+k+1)}\right)^2\ dx$$ Now interchanging the summation and integral (which needs to be justified) we get $$I_n=(n!)^2\int_{0}^{\infty}\sum_{k=1}^{\infty}\left(\frac{\Gamma(x+n+k)}{\Gamma(x+2n+k+1)}\right)^2\ dx$$ Now the infinite sum above is see here

$$I_n=(n!)^2\int_{0}^{\infty} \left(\frac{\Gamma(x+n+1)}{\Gamma(x+2n+2)}\right)^2{}_3F_2\left ( 1,x+n+1,x+n+1;x+2n+2,x+2n+2;1 \right)dx$$

where ${}_3F_2$ represents the hypergeometric function.

Thank you!

Edit: @Gary and @TymaGaidash have in their elegant answers found that $$\sum_{k=1}^{\infty}\int_{0}^{\infty}\operatorname B^2(x+n+k,n+1)\ dx =\binom{2n}n\gamma-\sum_{j=0}^n\binom nj^2(2(H_{n-j}-H_j)\ln((j+n)!)+H_{n+j})$$ So if we define$$a_n:= \sum_{j=0}^n\binom nj^2(2(H_{n-j}-H_j)\ln((j+n)!) $$ I need an asymptotic expansion for $a_n$ as $n\to \infty$. I believe that we can apply Laplace's method of asymptotic expansion of integrals (see here p.$322$)

Question: I need to prove that the limit $$ \lim_{n\to \infty} \frac{n\ 4^{2n}}{e^{2n}}\{-d_{2n} a_n\}\leq \frac{3}{4}$$ where $d_{2n}=\text{LCM}(1,2,...,2n)$, $\{x\}$ is the fractional part of $x$ and $a_n$ is defined as above.

Edit Can we at least simplify the above fractional part $\{-d_{2n} a_n\}$? Can we use the (anti) symmetry in $a_n$ to get a simpler form of $\{-d_{2n} a_n\}$?

I would really appreciate an answer which I can accept. Thank you!

Max
  • 879
  • The integrand is $\frac1{(x+n+k)_{n+1}}$, a Pochhammer symbol which has a finite sum representation after partial fractions, which allows us to take the integral as a finite sum of logarithms. However, it is hard to see a pattern for partial fraction coefficients. – Тyma Gaidash Jul 29 '23 at 14:58
  • 1
    @TymaGaidash Thanks a lot for your inputs – Max Jul 29 '23 at 15:02
  • @Max hi see https://mathworld.wolfram.com/DixonsTheorem.html . – Barackouda Aug 01 '23 at 09:30
  • @Max Sorry I haven't the skill to do that . Ask for Tyma Gaidash or Gary . – Barackouda Aug 01 '23 at 12:27
  • @Max For $\text{LCM}(1,2,\dots 2n)$, the $1$ does not fit the pattern, unless $n=\frac12$. Is it $\text{LCM}(1,2,4,6,8,\dots,2n)$? – Тyma Gaidash Aug 05 '23 at 12:04
  • @TymaGaidash Apologies. Actually $d_n$ is the LCM of first $n$ natural numbers and hence $d_{2n}$ is the LCM of first $2n$ natural numbers. – Max Aug 05 '23 at 12:09
  • Restore the question to an earlier version so that the answers below make sense. Currently they do not match the question. Accept the answer you think answers your question the best. After that, post your new problem as a new question on the site. – Gary Aug 05 '23 at 22:59
  • @Gary Sorry, but no answer is complete. – Max Aug 06 '23 at 01:23
  • 2
    @Max Your post starts "I need an asymptotic expansion/closed form for...". There are answers for that below. Later you write "I need an asymptotic expansion for $a_n$ as $n→∞$". There are answers for that too. If you find them incomplete, please indicate why you do so. The last question should be moved to a new question separate from this one. – Gary Aug 06 '23 at 01:29

6 Answers6

7

This is not a complete answer. Using the limit representation in Tyma Gaidash's answer, and the known results $$ H_{n + j + r} = \log (n + j + r) + \gamma + o(1), \quad r\to+\infty, $$ (with $\gamma$ being the Euler–Mascheroni constant) and $$ \sum\limits_{j = 0}^n {\binom{n}{j}^2 } = \binom{2n}{n}, $$ we can derive the explicit formula $$ \boxed{I_n = \binom{2n}{n}\gamma - \sum\limits_{j = 0}^n {\binom{n}{j}^2 (2(H_{n - j} - H_j )\log (( n+j)!) + H_{n + j} )} ,} $$ provided that $$ \mathop {\lim }\limits_{r \to + \infty } \sum\limits_{j = 0}^n {\binom{n}{j}^2 \left( {2(H_{n - j} - H_j )\log ((n + j + r)!) + \log (n + j + r)} \right)} = 0. $$ Denote the expression under the limit by $S_n(r)$. Performing the change of summation index $j\to n-j$ and taking the average with the original expression, we find $$ S_n (r) = \sum\limits_{j = 0}^n {\binom{n}{j}^2 \left( {(H_{n - j} - H_j )\log \frac{{(n + j + r)!}}{{(2n - j + r)!}} + \frac{1}{2}\log ((2n - j + r)(n + j + r))} \right)} . $$ Stirling's formula and the Maclaurin series of the logarithm then yields $$ S_n (r) = (\log r)\sum\limits_{j = 0}^n {\binom{n}{j}^2 \left( {(H_{n - j} - H_j )(2j - n) + 1} \right)} + o(1) $$ as $r\to+\infty$. Therefore, it remains to show that $$ \sum\limits_{j = 0}^n {\binom{n}{j}^2 ((H_{n - j} - H_j )(2j - n) + 1)} =0 $$ for all $n\ge 1$. Because of the symmetry in $H_{n - j} - H_j$, this may be further simplifed to the claim that $$ \sum\limits_{j = 0}^n {\binom{n}{j}^2 (2j(H_{n - j} - H_j )+ 1)} =0 $$ for all $n\ge 1$. Computer algebra software confirms this for $n=1,2,3\ldots,200$.

Addendum. The limit expression in the original version of Tyma Gaidash's answer was $$ I_n=\lim_{r\to\infty}\sum_{j=0}^n\binom n j^2\left(2 (H_{n-j}-H_j)\ln\left(\frac{(n+j+r)!}{(n+j)!}\right)+H_{n+j+r}-H_{n+j}\right). $$

Gary
  • 36,640
  • +1. Beautiful answer and well written. Can we show that $$ \sum\limits_{j = 0}^n {\binom{n}{j}^2 (2j(H_{n - j} - H_j )+ 1)} =0 $$ It seems to be true as you said for $n=1,2,3,...,200$. But how to show that it is $0$ for all $n$? – Max Jul 31 '23 at 04:18
  • @Max I have not succeeded in showing it for all $n\ge 1$ yet. That is why I say at the beginning that this is not a complete answer. – Gary Jul 31 '23 at 04:19
  • 1
    @Max You again totally modified your question by adding a new problem that does not follow from the previous ones in a straightforward manner. Post it as a separate question with explanation why you consider that particular limit. You should provide context. – Gary Aug 05 '23 at 08:51
  • Thanks for answering. I have accepted your answer. Its a request that please answer my question here or in the new question which I have asked here: https://math.stackexchange.com/questions/4746850/limit-involving-fractional-part-function – Max Aug 07 '23 at 16:04
5

Here are the asked details after applying the Thomae formula $$I_n=n!^2\int_0^{\infty}\Gamma(x+n+1)\frac{\Gamma(2n+2)}{\Gamma(2n+3)\Gamma(3n+3+x)}\, _3\,F_2(n+1,n+1,2n+2;3n+3+x,2n+3;1)dx$$

$$=\frac{n!^2}{2n+2}\int_0^{\infty}\frac{\Gamma(x+n+1)}{\Gamma(3n+3+x)}\sum_{k=0}^{\infty}\frac{1}{k!}\frac{((n+1)_k)^2(2n+2)_k}{(2n+3)_k(3n+3+x)_k}$$$$=n!^2\int_0^{\infty}\frac{\Gamma(x+n+1)}{\Gamma(3n+3+x)}\sum_{k=0}^{\infty}\frac{1}{k!}\frac{((n+1)_k)^2}{(2n+2+k)(3n+3+x)_k}dx$$ Up to the appearance of a $2n+2+k$ in the denominator, we get a $_2\, F_1(n+1,n+1; 3n+3+x;1)$ function. Therefore another presentation is $$I_n=n!^2\int_0^{\infty}\frac{\Gamma(x+n+1)}{\Gamma(3n+3+x)}\left(\int_0^1z^{2n+2}\, _2\,F_1(n+1,n+1; 3n+3+x;z)dz\right)dz$$ I may have done some errors of computation...

In the other hand, there are completely different ways of presentation of $I_n$ by replacing $\sum_{k=1}^{\infty}B(n+k+x,n+1)^2$ by $$\int_0^1\int_0^1(t^xt^n(1-t)^n))(s^xs^n(1-s)^n))\sum_{k=1}^{\infty}(ts)^{k-1}dtds$$$$=\int_0^1\int_0^1(t^xt^n(1-t)^n))(s^xs^n(1-s)^n))\frac{1}{1-ts}dtds.$$ This leads to $$I_n=\int_0^1\int_0^1\frac{(st(1-t)(1-s))^n}{(1-ts)(-\log st)}dtds.$$

5

$\def\H{\operatorname H}$ Applying partial fractions on $\frac{1}{(x(x+1)(x+2)\dots(x+n))^2}$ uses the harmonic numbers:

$$I_n=\sum_{k=1}^\infty\int_0^\infty\sum_{j=0}^n\binom nj^2\left(2\frac{\H_j-\H_{n-j}}{x+j+k+n}+\frac1{(x+j+k+n)^2}\right)dx$$

Integrating and evaluating the negative antiderivative at $x=0$:

$$I_n=\sum_{j=0}^n\binom n j^2\sum_{k=1}^\infty\left(2(\H_{n-j}-\H_j)\ln(j+k+n)+\frac1{j+k+n}\right)$$

@Gary sums over $k$ with the Euler-Mascheroni constant, to get:

$$\boxed{\sum_{k=1}^{\infty}\int_{0}^{\infty}\operatorname B^2(x+n+k,n+1)\ dx =\binom{2n}n\gamma-\sum_{j=0}^n\binom nj^2(2(\H_{n-j}-\H_j)\ln((j+n)!)+\H_{n+j})}$$

Results: $$I_1=2\gamma-\frac52+2\ln(2)\\I_2=6\gamma-\frac{131}{12}+6\ln(2)+3\ln(3)\\I_3=20\gamma-\frac{523}{12}+\frac{11}3\ln(4)+\frac{38}3\ln(5)+\frac{11}3\ln(6)\\I_4=70\gamma-\frac{143657}{840}+\frac{25}6\ln(5)+\frac{185}6\ln(6)+\frac{185}6\ln(7)+\frac{25}6\ln(8)\\I_5=252\gamma-\frac{168401}{252}+\frac{137}6\ln(6)+\frac{881}{15}\ln(7)+\frac{627}5\ln(8)+\frac{881}{15}\ln(9)+\frac{137}{30}\ln(10)\\I_6=924\gamma-\frac{14468369}{5544}+\frac{49}{10}\ln(7)+\frac{973}{10}\ln(8)+\frac{1799}5\ln(9)+\frac{1799}5\ln(10)+\frac{973}{10}\ln(11)+\frac{49}{10}\ln(12)\\\vdots$$

Тyma Gaidash
  • 13,576
  • +1 for your answer. Please edit your answer for general $n$ – Max Jul 30 '23 at 18:30
  • 1
    $$ I_n = \binom{2n}{n}\gamma - \sum\limits_{j = 0}^n {\binom{n}{j}^2 (2(H_{n - j} - H_j )\log ((j + n)!) + H_{n + j} )} $$ – Gary Jul 31 '23 at 00:10
  • If you replace the upper limit of integration by $r$ and put a $\lim_{r\to+\infty}$ in the front, you will end up with the formula at the bottom of my incomplete answer. – Gary Jul 31 '23 at 04:51
5

This is just to add to the nice posted solutions the heuristic approach for finding the asymptotics of $I_n$ at $n\to\infty$. We can use the approach by @Gérard Letac and present $I_n$ as $$I_n=\sum_{k=1}^{\infty}\int_{0}^{\infty}(B(x+n+k,n+1))^2\ dx=\int_0^1\int_0^1\frac{\big(st(1-t)(1-s)\big)^n}{(1-ts)(-\log st)}dtds$$ Making the substitutions $s=x+\frac12;\,t=y+\frac12$ $$I_n=\int_{-1/2}^{1/2}\int_{-1/2}^{1/2}\frac{\big(\frac14-x^2\big)^n\big(\frac14-y^2\big)^n}{\big(\frac34-\frac x2-\frac y2-xy\big)\left(-\ln\big(\frac14+\frac x2+\frac y2+xy\big)\right)}dxdy$$ $$=\frac1{4^{2n}}\int_{-1}^1\int_{-1}^1\frac{(1-x^2)^n(1-y^2)^n}{(3-x-y-xy)\big(\ln 4-\ln(1+x+y+xy)\big)}dxdy\tag{1}$$ At $n\to\infty$ the integral gets the main contribution at small $\,x,\,y\,$; for the main asymptotic term we get $$I_n\sim\frac1{4^{2n}6\ln2}\int_{-1}^1\int_{-1}^1(1-x^2)^n(1-y^2)^ndxdy\sim\frac\pi{6\ln2}\frac{4^{-2n}}n$$ We can also get higher asymptotic terms - just decomposing the integrand in (1) near $\,x,\,y =0$ , and to make sure that $$\boxed{\,\,I_n=\frac\pi{6\ln2}\frac{4^{-2n}}n\left(1+\frac1n\Big(\frac1{4\ln^22}-\frac1{12\ln2}-\frac{23}{36}\Big)+O\left(\frac1{n^2}\right)\right)\,\,}$$ $\bf{Addendum}$

Asymptotics of $a_n:= \sum_{j=0}^n\binom nj^2(2(H_{n-j}-H_j)\ln((j+n)!)$

The answer was obtained by @Gary; I just want to add a bit different way of calculation by means of using Stirling's formula $$I_n=\binom{2n}n\gamma-\sum_{j=0}^n\binom nj^2(2(H_{n-j}-H_j)\ln((j+n)!)+H_{n+j})=\binom{2n}n\gamma-a_n-b_n$$ where we denoted $b_n=\sum_{j=0}^n\binom nj^2H_{n+j}$

As $\binom nj^2$ reaches a sharp maximum near $j=\frac n2$, we can choose $n$ even for a while, $j=\frac n2+k $ and present $b_n$ as $$b_n=\sum_{k=-n/2}^{n/2}\left(\frac{n!}{\big(\frac n2-k\big)!\big(\frac n2+k\big)!}\right)^2H_{\frac{3n}2+k}$$ Using the Stirling's formula for $p!$ ( for $p\gg1$) $$b_n\sim\sum_{k=-n/2}^{n/2}\left(\frac{\sqrt{2\pi n}\left(\frac ne\right)^n}{\sqrt{2\pi(\frac n2+k)}\sqrt{2\pi(\frac n2-k)}\Big(\frac{\frac n2+k}e\Big)^{\frac n2+k}\Big(\frac{\frac n2-k}e\Big)^{\frac n2-k}}\right)^2H_{\frac{3n}2+k}$$ The terms decline sharply as soon as $k$ excides $\sqrt n\,$ , so we can switch from summation to integration. Given that $H_{\frac{3n}2+k}$ is slowly changing function, we are allowed just to take its value at $k=0$ and use the asymptotics $H_\frac{3n}2=\gamma+\ln\frac{3n}2+O\big(\frac1n\big)$

After straightforward manipulations we get $$b_n\sim\frac{2\cdot4^n\big(\gamma+\ln\frac32+\ln n\big)}{\pi\sqrt n}\int_{-\infty}^\infty e^{-4t^2}dt=\frac{4^n}{\sqrt{\pi n}}\left(\gamma+\ln\frac32+\ln n\right)$$ Due to the fact that $$\binom{2n}n\sim \frac{4^n}{\sqrt{\pi n}}\to\infty\,\,\text{at}\,\,n\to\infty$$ and $I_n\to0$ at $n\to\infty$, we conclude that $$\boxed{\,\,a_n\sim-\frac{ 4^n}{\sqrt{\pi n}}\left(\ln\frac32+\ln n\right)\,\,}$$

Svyatoslav
  • 20,502
  • 1
    (+1) Numerically confirmed. – Gary Aug 01 '23 at 23:24
  • 2
    Instead of editing the question over and over again and adding new problems, please ask a new question on the site and refer back to this one. – Gary Aug 02 '23 at 21:51
3

This answers the question about the asymptotics of $a_n$ provided that the conjectured formula for $I_n$ in my other answer is correct. Note that since $H_k=\psi(k+1)+\gamma$, we can write $$ a_n = - I_n - \sum\limits_{j = 0}^n {\binom{n}{j}^2 \psi (n + j + 1)} = - \sum\limits_{j = 0}^n {\binom{n}{j}^2 \psi (n + j + 1)} + o(1). $$ The $o$-term follows from Svyatoslav's answer. Now by the asymptotic result $\psi(k+1)=\log k +\mathcal{O}(k^{-1})$, we have \begin{align*} \sum\limits_{j = 0}^n {\binom{n}{j}^2 \psi (n + j + 1)} & = \sum\limits_{j = 0}^n {\binom{n}{j}^2 \log (n + j)} + \mathcal{O}(1)\sum\limits_{j = 0}^n {\binom{n}{j}^2 \frac{1}{{n + j + 1}}} \\ & = \sum\limits_{j = 0}^n {\binom{n}{j}^2 \log (n + j)} + \mathcal{O}\!\left( {\frac{1}{n}} \right)\binom{2n}{n}. \end{align*} Since $\log n \le \log(n+j)\le \log n+\log 2$, it follows that $$ a_n \sim -\binom{2n}{n}\log n $$ as $n\to+\infty$.

Addendum. Performing the change of summation index $j\to n−j$ and taking the average with the original expression, we find $$ \sum\limits_{j = 0}^n {\binom{n}{j}^2 \log (n + j)} = \sum\limits_{j = 0}^n {\binom{n}{j}^2 \log \sqrt {(n + j)(2n - j)} } . $$ Now \begin{align*} \log \sqrt {(n + j)(2n - j)} & = \log \left( {\frac{{3n}}{2}} \right) + \log \sqrt {1 - \frac{4}{9}\left( {\frac{1}{2} - \frac{j}{n}} \right)^2 } \\ & = \log \left( {\frac{{3n}}{2}} \right) + \mathcal{O}(1)\left( {\frac{1}{2} - \frac{j}{n}} \right)^2 . \end{align*} Numerics suggest that $$ \sum\limits_{j = 0}^n {\binom{n}{j}^2 \left( {\frac{1}{2} - \frac{j}{n}} \right)^2 } \sim \frac{1}{{8n}}\binom{2n}{n}, $$ as $n\to+\infty$. This would lead to the more precise result $$ a_n = -\binom{2n}{n}\left(\log \left( {\frac{{3n}}{2}} \right) +\mathcal{O}\!\left( {\frac{1}{n}} \right)\right) $$ as $n\to+\infty$.

Gary
  • 36,640
0

A way to simplify $_3\,F_2$ is to use the Thomae formula which says

$$s(A,B,C,D,E)=\frac{\Gamma(C)}{\Gamma(D)\Gamma(E)}_3\,F_2(A,B,C;D,E;1)= $$ $$s(D-C,E-C,D+E-A-B-C, D+E-A-C,D+D-B-C)$$ In your integral formula using $A=1, \ B=x+n+1, $ etc, the new $_3\,F_2$ seems to contain $x$ only in $D+E-A-C.$ I became aware of this formula while writing

Letac, G. and Piccioni, M. (2012) 'Random continued fractions with beta hypergeometric distribution. Ann. Probab. vol 40, number 3, 1105-1134

containing all references and our proof of the Thomae formula.